Use non-breaking space (~) between 'Figure' and the figure number.
[course.git] / latex / problems / Serway_and_Jewett_8 / problem05.63.tex
1 \begin{problem*}{5.63}
2 A crate of wieght $F_g$ is pushed by a force $\vect{P}$ on a
3 horizontal floor as shown in Figure~P5.63.  The coefficient of static
4 friction is $\mu_s$, and $\vect{P}$ is directed at an angle $\theta$
5 below the horizontal.  \Part{a} Show that the minimum value of $P$
6 that will move the crate is given by
7 \begin{equation}
8   P = \frac{\mu_s F_g \sec\theta}{1 - \mu_s \tan\theta}
9 \end{equation}
10 \Part{b} Find the condition on $\theta$ in terms of $\mu_s$, for
11 which motion of the crate is impossible for any value of $P$.
12 \begin{center}
13 \begin{asy}
14 import Mechanics;
15
16 real u = 1cm;
17 pair p = (0.3u, -0.3u);
18
19 Surface s = Surface((-p.x,0), (1u+p.x,0));
20 s.draw();
21 Block b = Block((0.5u, 0.35u), width=1u, height=0.7u, "crate");
22 b.draw();
23 Vector P = Force((0,0.7u)-p, mag=length(p), dir=degrees(p), "$\vect{P}$");
24 P.draw();
25 \end{asy}
26 \end{center}
27 \end{problem*}
28
29 \begin{solution}
30 \Part{a}
31 The normal force must resist both the force of gravity and the
32 vertical component of $\vect{P}$, so
33 \begin{equation}
34   N = F_g + P\sin(\theta) \;.
35 \end{equation}
36 This moves the crate when the horizontal component of $\vect{P}$
37 balances the force of friction.
38 \begin{align}
39   P\cos(\theta) &= \mu_s N = \mu_s (F_g + P\sin(\theta)) \\
40   P(\cos(\theta) - \mu_s \sin(\theta)) &= \mu_s F_g \\
41   P(1 - \mu_s \tan(\theta)) &= \mu_s F_g \sec(\theta) \\
42   P &= \ans{\frac{\mu_s F_g \sec(\theta)}{1 - \mu_s \tan(\theta)}} \;,
43 \end{align}
44 which is what we set out to show.
45
46 Note that this formula is only valid when there is an actual normal
47 force to provide friction.  Therefore $P\cos(\theta) > 0$.  We can
48 posit, without loss of generality, that $P>0$, in which case the
49 restriction is $-90\dg < \theta < 90\dg$.  By symmetry, the situation
50 for the backside $180\dg$ is just a mirror image of the frontside.
51
52 \Part{b}
53 As $P$ becomes larger, the $F_g$ component of our horizontal force
54 balance becomes negligable, so we cannot move the block when
55 \begin{align}
56   P\cos(\theta) &\le \mu_s P \sin(\theta) \\
57   \frac{1}{\mu_s} &\le \tan(\theta) \\
58   \theta &\ge \ans{\arctan\p({\frac{1}{\mu_s}})} \equiv \theta_c \;,
59 \end{align}
60 where the last step uses the fact that $\tan(\theta)$ is strictly
61 increasing on the range $\theta\in(-90\dg,90\dg)$.
62
63 What does this mean about our answer to \Part{a}?  Let's rework the
64 condition to look more like the denominator in the \Part{a} answer.
65 \begin{align}
66   \tan(\theta) &\ge \frac{1}{\mu_s} \\
67   0 &\ge \frac{1}{\mu_s} - \tan(\theta) \\
68   0 &\ge 1 - \mu_s \tan(\theta) \;,
69 \end{align}
70 so the denominator is negative or zero for $\theta \ge \theta_c $.
71 For $\theta$ just below the cutoff, the denominator is small but
72 positive, and you get a really large value for $P$.  For
73 $\theta=\theta_c$, the denominator is zero, and you get an infinite
74 value for $P$.  For $\theta$ above the cutoff, the denominator is
75 negative, so $P$ is also negative, which, as I pointed out
76 in \Part{a}, is not allowed.
77
78 The whole thing is a bit easier to understand if we rephrase the
79 answer to \Part{a} as
80 \begin{equation}
81   P = \frac{\mu_s F_g}{\cos(\theta) - \mu_s \sin(\theta)}
82     = \frac{C}{\cos(\theta) - \mu_s \sin(\theta)}
83     = (A\cos(\theta) - B\sin(\theta))^{-1} \;,
84 \end{equation}
85 where $C=\equiv \mu_s F_g$, $A\equiv 1/C$, and $B\equiv \mu_s/C=1/F_g$.
86 We can consolidate to a single trig term using
87 \begin{align}
88   \sin(a \pm b) &= \sin(a)\cos(b) \pm \cos(a)\sin(b) \\
89   (D\sin(a - b))^{-1} &= (D \sin(a)\cos(b) - D\cos(a)\sin(b))^{-1} \;.
90 \end{align}
91 Matching with our formula,
92 \begin{align}
93   \theta &= b \\
94   A &= D\sin(a) \\
95   B &= D\cos(a) \\
96   \tan(a) &= \frac{A}{B} = \frac{1/C}{\mu_s/C} = \frac{1}{\mu_s} \\
97   a &= \arctan\p({\frac{1}{\mu_s}}) \\
98   D &= \frac{B}{\cos(a)}
99     = \frac{B}{\cos\p({\arctan\p({\frac{1}{\mu_s}})})}
100     = B\sqrt{1+\frac{1}{\mu_s^2}}
101     = \frac{\sqrt{1+\frac{1}{\mu_s^2}}}{F_g} \\
102   P &= D^{-1} \p({ \sin(a - b) })^{-1}
103     = \frac{F_g}{\sqrt{1 + \frac{1}{\mu_s^2}}}
104       \csc\p({\arctan\p({\frac{1}{\mu_s}}) - \theta}) \;.
105 \end{align}
106 This doesn't look as clean as the phrasing in \Part{a}, but it makes
107 the dependence of $P$ on $\theta$ much clearer.  For example, $P$ is
108 obviously negative for $\theta > \theta_c \equiv \arctan(1/\mu_s)$.
109 The dependency on $\theta$ over the rest of the range is
110 \begin{equation}
111   P \propto \csc(\theta_c - \theta) = \frac{1}{\sin(\theta_c - \theta)}
112 \end{equation}
113 Because $\mu_s$ is a positive number, $1/\mu_s$ will also be positive,
114 and $\theta_c$ will be between $0$ and $90\dg$.  The status on all
115 possible angles looks something like
116 \begin{center}
117 \begin{asy}
118 import Mechanics;
119
120 real tc = 67;
121 real u = 1cm;
122 real r = u;
123 real R = 1.5u;
124 real L = 3u;
125
126 Angle Atc = Angle((1,0), (0,0), dir(tc), "$\theta_c$");
127 Atc.draw();
128
129 draw(scale(r)*unitcircle);
130 draw((-R,0)--(R,0));
131 draw((0,-R)--(0,R), red);
132 draw((-dir(tc)*R)--(dir(tc)*R), blue);
133
134 label("$\cos(\theta)>0$", L*dir(0), red);
135 label("$P > 0$", L*dir(tc-90), blue);
136 \end{asy}
137 \end{center}
138 Taking the $\cos(\theta)>0$ portion of our $P$ dependence (where the
139 equation we started with in \Part{a} applies, and combining it with
140 the reflection (which applies when $\cos(\theta)<0$, we get
141 \begin{equation}
142   P = \begin{cases}
143         \infty & \text{if $\theta_c \ge \theta < 180\dg-\theta_c$} \\
144         \frac{F_g}{\sqrt{1 + \frac{1}{\mu_s^2}}}
145           \csc\p({\arctan\p({\frac{1}{\mu_s}}) - \theta})
146           & \text{if $-90\dg \le \theta < \theta_c$} \\
147         \frac{F_g}{\sqrt{1 + \frac{1}{\mu_s^2}}}
148           \csc\p({\arctan\p({\frac{1}{\mu_s}}) - 180\dg + \theta})
149           & \text{if $180\dg - \theta_c < \theta \le 180\dg$
150                   or $-180\dg \le \theta \le -90\dg$} \\
151       \end{cases}
152 \end{equation}
153 which looks like
154 \begin{center}
155 \begin{asy}
156 import graph;
157 import Mechanics;
158
159 real u = 2cm;
160
161 //size(0, 2u);
162 scale(false);
163
164 real tc = 67;
165 real tc_r = tc*pi/180;         // theta_c in radians
166 real tc_buf = (tc-25)*pi/180;  // weakened version of tc_r
167
168 real r_scale(real r) { return u/2 * r; }
169 real R(real theta) { return r_scale( 1/sin(tc_r-theta) ); }
170 real L(real theta) { return R(pi-theta); }
171 real onef (real theta) { return r_scale(1); }
172 real twof (real theta) { return r_scale(2); }
173
174 draw((0,0)--(u,0));
175 draw((0,0)--(u*dir(tc)));
176 draw((0,0)--(u*dir(180-tc)));
177
178 draw(polargraph(onef, -pi-tc_r, tc_r));
179 draw(polargraph(twof, -pi-tc_r, tc_r));
180 label("$1$", r_scale(1)*dir(tc-90), SE);
181 label("$2$", r_scale(2)*dir(tc-90), SE);
182
183 draw(polargraph(R, -pi/2, tc_buf), blue);
184 draw(polargraph(L, pi-tc_buf, 3pi/2), blue);
185
186 Angle Ainf = Angle(dir(180-tc), (0,0), dir(tc), radius=0.7u, red, "$\infty$");
187 Ainf.draw();
188 Angle Atc = Angle(dir(0), (0,0), dir(tc), "$\theta_c$");
189 Atc.draw();
190 \end{asy}
191 \end{center}
192 where I've just plotted the $\theta$ dependence of $P$, setting the
193 constant $F_g/\sqrt{ }$ term equal to $1$.  Note that the $\csc$ makes
194 nice, straight lines in this polar plot.
195 \end{solution}